Difference between revisions of "2006 AMC 12A Problems/Problem 25"

Line 7: Line 7:
 
<math>(2)</math>  If <math>S</math> contains <math>k</math> elements, then <math>S</math> contains no number less than <math>k</math>.
 
<math>(2)</math>  If <math>S</math> contains <math>k</math> elements, then <math>S</math> contains no number less than <math>k</math>.
  
<math> \mathrm{(A) \ } 277\qquad \mathrm{(B) \ } 311\qquad \mathrm{(C) \ } 376\qquad \mathrm{(D) \ } 377\qquad \mathrm{(E) \ }  405</math>
+
<math> \mathrm{(A) \ } 277\qquad \mathrm{(B) \ } 311\qquad \mathrm{(C) \ } 376\qquad \mathrm{(D) \ } 377</math><math>\mathrm{(E) \ }  405</math>
  
 
== Solution ==
 
== Solution ==

Revision as of 00:13, 11 July 2006

Problem

How many non-empty subsets $S$ of $\{1,2,3,\ldots ,15\}$ have the following two properties?

$(1)$ No two consecutive integers belong to $S$.

$(2)$ If $S$ contains $k$ elements, then $S$ contains no number less than $k$.

$\mathrm{(A) \ } 277\qquad \mathrm{(B) \ } 311\qquad \mathrm{(C) \ } 376\qquad \mathrm{(D) \ } 377$$\mathrm{(E) \ }  405$

Solution

See also